Merge remote branch 'public/master'
[course.git] / latex / problems / Serway_and_Jewett_4 / problem22.06.tex
1 \begin{problem*}{22.6}
2 A proton moves with a velocity of $\vect{v} = (2\ihat -4\jhat
3 +\khat)\U{m/s}$ in a region in which the magnetic field is $\vect{B} =
4 (\ihat + 2\jhat -3\khat)\U{T}$.  What is the magnitude of the magnetic
5 force this charge experiences?
6 \end{problem*} % problem 22.6
7
8 \begin{solution}
9 \begin{align}
10  \vect{F} &= q \vect{v}\times\vect{B}
11    = q  \left|
12         \begin{matrix}
13          \ihat & \jhat & \khat \\
14          2 & -4 & 1 \\
15          1 & 2 & -3
16         \end{matrix}
17         \right|
18    = q [\ihat(12-2) -\jhat(-6-1) +\khat(4-(-4))]
19    = q (10\ihat+7\jhat+8\khat) \\
20  |\vect{F}| &= q \sqrt{10^2+7^2+8^2}
21              = \ans{2.34\E{-18}\U{N}}
22 \end{align}
23 \end{solution}